set of all maximum points is either finite or countable












1












$begingroup$




Try:



Call $M$ to be the set of all maximum points of a given $f: mathbb{R} to mathbb{R}$. We wish to construct $phi: mathbb{N} to M $ so that $phi$ is bijective.



Notice that $x_i$ is maximum of $f$ is we can find $epsilon_i$ so that ....



Maybe we can define $phi(i) = x_i $ and check for bijectivity.



If $phi(i) = phi(k) implies x_i = x_k implies i=k $



Now, any points $x_j$ where $f$ attains its maximum there corresponds an index $j$ so that surjectivity follows and we have bijectivity.



Thus, $M$ is finite our countable.



Is my approach correct?










share|cite|improve this question









$endgroup$












  • $begingroup$
    Note that the definition allows you to associate to each maximum point an open interval having a certain property such that this property implies that the open intervals are pairwise disjoint. Thus, it is enough to prove that any collection of pairwise disjoint open intervals is at most countable, a result that you might already have available, but if not, think about what you can deduce if you pick a rational number from each of these open intervals (you'll get at most countably many rationals, no two of which belong to the same open interval . . .).
    $endgroup$
    – Dave L. Renfro
    Jan 24 at 9:02


















1












$begingroup$




Try:



Call $M$ to be the set of all maximum points of a given $f: mathbb{R} to mathbb{R}$. We wish to construct $phi: mathbb{N} to M $ so that $phi$ is bijective.



Notice that $x_i$ is maximum of $f$ is we can find $epsilon_i$ so that ....



Maybe we can define $phi(i) = x_i $ and check for bijectivity.



If $phi(i) = phi(k) implies x_i = x_k implies i=k $



Now, any points $x_j$ where $f$ attains its maximum there corresponds an index $j$ so that surjectivity follows and we have bijectivity.



Thus, $M$ is finite our countable.



Is my approach correct?










share|cite|improve this question









$endgroup$












  • $begingroup$
    Note that the definition allows you to associate to each maximum point an open interval having a certain property such that this property implies that the open intervals are pairwise disjoint. Thus, it is enough to prove that any collection of pairwise disjoint open intervals is at most countable, a result that you might already have available, but if not, think about what you can deduce if you pick a rational number from each of these open intervals (you'll get at most countably many rationals, no two of which belong to the same open interval . . .).
    $endgroup$
    – Dave L. Renfro
    Jan 24 at 9:02
















1












1








1





$begingroup$




Try:



Call $M$ to be the set of all maximum points of a given $f: mathbb{R} to mathbb{R}$. We wish to construct $phi: mathbb{N} to M $ so that $phi$ is bijective.



Notice that $x_i$ is maximum of $f$ is we can find $epsilon_i$ so that ....



Maybe we can define $phi(i) = x_i $ and check for bijectivity.



If $phi(i) = phi(k) implies x_i = x_k implies i=k $



Now, any points $x_j$ where $f$ attains its maximum there corresponds an index $j$ so that surjectivity follows and we have bijectivity.



Thus, $M$ is finite our countable.



Is my approach correct?










share|cite|improve this question









$endgroup$






Try:



Call $M$ to be the set of all maximum points of a given $f: mathbb{R} to mathbb{R}$. We wish to construct $phi: mathbb{N} to M $ so that $phi$ is bijective.



Notice that $x_i$ is maximum of $f$ is we can find $epsilon_i$ so that ....



Maybe we can define $phi(i) = x_i $ and check for bijectivity.



If $phi(i) = phi(k) implies x_i = x_k implies i=k $



Now, any points $x_j$ where $f$ attains its maximum there corresponds an index $j$ so that surjectivity follows and we have bijectivity.



Thus, $M$ is finite our countable.



Is my approach correct?







real-analysis calculus






share|cite|improve this question













share|cite|improve this question











share|cite|improve this question




share|cite|improve this question










asked Jan 24 at 2:38









Jimmy SabaterJimmy Sabater

3,023325




3,023325












  • $begingroup$
    Note that the definition allows you to associate to each maximum point an open interval having a certain property such that this property implies that the open intervals are pairwise disjoint. Thus, it is enough to prove that any collection of pairwise disjoint open intervals is at most countable, a result that you might already have available, but if not, think about what you can deduce if you pick a rational number from each of these open intervals (you'll get at most countably many rationals, no two of which belong to the same open interval . . .).
    $endgroup$
    – Dave L. Renfro
    Jan 24 at 9:02




















  • $begingroup$
    Note that the definition allows you to associate to each maximum point an open interval having a certain property such that this property implies that the open intervals are pairwise disjoint. Thus, it is enough to prove that any collection of pairwise disjoint open intervals is at most countable, a result that you might already have available, but if not, think about what you can deduce if you pick a rational number from each of these open intervals (you'll get at most countably many rationals, no two of which belong to the same open interval . . .).
    $endgroup$
    – Dave L. Renfro
    Jan 24 at 9:02


















$begingroup$
Note that the definition allows you to associate to each maximum point an open interval having a certain property such that this property implies that the open intervals are pairwise disjoint. Thus, it is enough to prove that any collection of pairwise disjoint open intervals is at most countable, a result that you might already have available, but if not, think about what you can deduce if you pick a rational number from each of these open intervals (you'll get at most countably many rationals, no two of which belong to the same open interval . . .).
$endgroup$
– Dave L. Renfro
Jan 24 at 9:02






$begingroup$
Note that the definition allows you to associate to each maximum point an open interval having a certain property such that this property implies that the open intervals are pairwise disjoint. Thus, it is enough to prove that any collection of pairwise disjoint open intervals is at most countable, a result that you might already have available, but if not, think about what you can deduce if you pick a rational number from each of these open intervals (you'll get at most countably many rationals, no two of which belong to the same open interval . . .).
$endgroup$
– Dave L. Renfro
Jan 24 at 9:02












1 Answer
1






active

oldest

votes


















1












$begingroup$

No. When you say that $x_i$ is a maximum point, you're assuming that such points are countable.



You can start by proving that the each maximum point is isolated and then use the fact that discrete subsets of $mathbb{R}$ are at most countable.






share|cite|improve this answer









$endgroup$













    Your Answer





    StackExchange.ifUsing("editor", function () {
    return StackExchange.using("mathjaxEditing", function () {
    StackExchange.MarkdownEditor.creationCallbacks.add(function (editor, postfix) {
    StackExchange.mathjaxEditing.prepareWmdForMathJax(editor, postfix, [["$", "$"], ["\\(","\\)"]]);
    });
    });
    }, "mathjax-editing");

    StackExchange.ready(function() {
    var channelOptions = {
    tags: "".split(" "),
    id: "69"
    };
    initTagRenderer("".split(" "), "".split(" "), channelOptions);

    StackExchange.using("externalEditor", function() {
    // Have to fire editor after snippets, if snippets enabled
    if (StackExchange.settings.snippets.snippetsEnabled) {
    StackExchange.using("snippets", function() {
    createEditor();
    });
    }
    else {
    createEditor();
    }
    });

    function createEditor() {
    StackExchange.prepareEditor({
    heartbeatType: 'answer',
    autoActivateHeartbeat: false,
    convertImagesToLinks: true,
    noModals: true,
    showLowRepImageUploadWarning: true,
    reputationToPostImages: 10,
    bindNavPrevention: true,
    postfix: "",
    imageUploader: {
    brandingHtml: "Powered by u003ca class="icon-imgur-white" href="https://imgur.com/"u003eu003c/au003e",
    contentPolicyHtml: "User contributions licensed under u003ca href="https://creativecommons.org/licenses/by-sa/3.0/"u003ecc by-sa 3.0 with attribution requiredu003c/au003e u003ca href="https://stackoverflow.com/legal/content-policy"u003e(content policy)u003c/au003e",
    allowUrls: true
    },
    noCode: true, onDemand: true,
    discardSelector: ".discard-answer"
    ,immediatelyShowMarkdownHelp:true
    });


    }
    });














    draft saved

    draft discarded


















    StackExchange.ready(
    function () {
    StackExchange.openid.initPostLogin('.new-post-login', 'https%3a%2f%2fmath.stackexchange.com%2fquestions%2f3085375%2fset-of-all-maximum-points-is-either-finite-or-countable%23new-answer', 'question_page');
    }
    );

    Post as a guest















    Required, but never shown

























    1 Answer
    1






    active

    oldest

    votes








    1 Answer
    1






    active

    oldest

    votes









    active

    oldest

    votes






    active

    oldest

    votes









    1












    $begingroup$

    No. When you say that $x_i$ is a maximum point, you're assuming that such points are countable.



    You can start by proving that the each maximum point is isolated and then use the fact that discrete subsets of $mathbb{R}$ are at most countable.






    share|cite|improve this answer









    $endgroup$


















      1












      $begingroup$

      No. When you say that $x_i$ is a maximum point, you're assuming that such points are countable.



      You can start by proving that the each maximum point is isolated and then use the fact that discrete subsets of $mathbb{R}$ are at most countable.






      share|cite|improve this answer









      $endgroup$
















        1












        1








        1





        $begingroup$

        No. When you say that $x_i$ is a maximum point, you're assuming that such points are countable.



        You can start by proving that the each maximum point is isolated and then use the fact that discrete subsets of $mathbb{R}$ are at most countable.






        share|cite|improve this answer









        $endgroup$



        No. When you say that $x_i$ is a maximum point, you're assuming that such points are countable.



        You can start by proving that the each maximum point is isolated and then use the fact that discrete subsets of $mathbb{R}$ are at most countable.







        share|cite|improve this answer












        share|cite|improve this answer



        share|cite|improve this answer










        answered Jan 24 at 3:49









        lzralbulzralbu

        640512




        640512






























            draft saved

            draft discarded




















































            Thanks for contributing an answer to Mathematics Stack Exchange!


            • Please be sure to answer the question. Provide details and share your research!

            But avoid



            • Asking for help, clarification, or responding to other answers.

            • Making statements based on opinion; back them up with references or personal experience.


            Use MathJax to format equations. MathJax reference.


            To learn more, see our tips on writing great answers.




            draft saved


            draft discarded














            StackExchange.ready(
            function () {
            StackExchange.openid.initPostLogin('.new-post-login', 'https%3a%2f%2fmath.stackexchange.com%2fquestions%2f3085375%2fset-of-all-maximum-points-is-either-finite-or-countable%23new-answer', 'question_page');
            }
            );

            Post as a guest















            Required, but never shown





















































            Required, but never shown














            Required, but never shown












            Required, but never shown







            Required, but never shown

































            Required, but never shown














            Required, but never shown












            Required, but never shown







            Required, but never shown







            Popular posts from this blog

            Can a sorcerer learn a 5th-level spell early by creating spell slots using the Font of Magic feature?

            Does disintegrating a polymorphed enemy still kill it after the 2018 errata?

            A Topological Invariant for $pi_3(U(n))$